Suppose the area of a trapezoid is 126 yd?. if the bases of the trapezoid are 17 yd and 11 yd long, what is the height?
a 4.5 yd
b. 9 yd
c. 2.25 yd
d. 18 yd

Answers

Answer 1

The height of the trapezoid is 9 yards. Therefore, the correct answer is option b. 9 yd.

To find the height of the trapezoid with the given area and base lengths, we will use the formula for the area of a trapezoid:

Area = (1/2) * (base1 + base2) * height

Here, the area is given as 126 square yards, base1 is 17 yards, and base2 is 11 yards. We need to find the height.

1. Substitute the given values into the formula:

126 = (1/2) * (17 + 11) * height

2. Simplify the equation:

126 = (1/2) * 28 * height

3. To isolate the height, divide both sides by (1/2) * 28:

height = 126 / ((1/2) * 28)

4. Calculate the result:

height = 126 / 14

height = 9

For more about height:

https://brainly.com/question/10726356

#SPJ4


Related Questions

Brody is going to invest $350 and leave it in an account for 18 years. Assuming the interest is compounded daily, what interest rate, to the


neatest tenth of a percent, would be required in order for Brody to end up with $790?

Answers

If the interest is compounded daily, the interest rate is 4.5%.

How to find the interest rate?

To determine the interest rate, we can use the compound interest formula:

A = P(1 + r/n)^(nt)

Where:

A = the final amount, $790

P = the principal, $350

r = the interest rate

n = the number of times the interest is compounded per year, in this case daily (n = 365)

t = the time period in years, 18

Substituting the values :

790 = 350(1 + r/365)³⁶⁵ˣ¹⁸

790 = 350(1 + r/365)⁶⁵⁷⁰

790/350 = (1 + r/365)⁶⁵⁷⁰

ln(790/350) = 6570 * ln (1 + r/365)

Using the property of logarithms that ln(1 + x) ~ x for small values of x, we can approximate the right-hand side as:

[ln(790/350)]/6570 = r/365

r = 0.045

r = 4.5%

Learn more about compound interest on:

https://brainly.com/question/24274034

#SPJ1

y varies inversely as x. y= 27 when x=5 Find y when x=3

Answers

As y varies inversely as x, the value of y when x = 3 is 45.

What is the value of y when x = 3?

Inverse proportionality is expressed as:

y ∝ 1/x

Hence:

y = k/x

Where k is the constant of proportionality.

First, we determine the constant of proportionality.

Using the information given in the problem.

When x = 5, y = 27

Substituting these values into the formula, we get:

y = k/x

27 = k/5

k = 135

Now that we have found the value of k, we can use the formula to find y when x = 3. Substituting x = 3 and k = 135, we get:

y = k/x

y = 135/3

y = 45

Therefore, the value of y is 45.

Learn more about proportionality here: https://brainly.com/question/27530069

#SPJ1

Monica deposits ​$ 300 into a savings account that pays a simple interest rate of ​ 3.4%. Paul deposits ​$400 into a savings account that pays a simple interest rate of ​3.3 %. Monica says that she will earn more interest in 1 year because her interest rate is higher. Is she​ correct? Justify your response.

Answers

Monica's claim is incorrect. Even though her interest rate is higher, she will earn less interest, $10.20, after one year than Paul because her initial deposit is lower.

Paul will earn more interest of $13.20 because he deposited more money, even though his interest rate is slightly lower.

To determine who will earn more interest in one year, we shall calculate the interest earned by each person using the simple interest formula.

What is the simple interest formula?

The formula for simple interest is:

I = P * r * t

where:

I = the interest earned

P = the principal (the amount deposited)

r = the interest rate (as a decimal)

t =s the time (in years)

For Monica, we are given:

P = $300

r = 0.034 (the interest rate is 3.4%)

t = 1 (the interest earned in one year)

Plugging these values, we have:

I = 300 * 0.034 * 1 = $10.20

So, Monica will earn $10.20 in interest after one year.

For Paul, we are given:

P = $400

r = 0.033 (interest rate = 3.3%)

t = 1 (interest earned in one year)

Plugging the values, we get:

I = 400 * 0.033 * 1 = $13.20

So, Paul will earn $13.20 in interest after one year.

Therefore, Monica's claim is incorrect. Monica will earn $10.20 in interest after one year while Paul will earn $13.20 in interest after one year.

Learn more about simple interest at brainly.com/question/25845758

#SPJ1

Find the measure of the arc or angle indicated. Assume that lines which appear tangent are tangent

Answers

The sum of all the angles in a quadrilateral is 360° ,So The angle measure indicated is 137°.

What is radius?

In classical geometry, the radius of a circle or sphere is any line segment that links the object's centre to its edge; in more modern usage, the term also refers to the length of such line segments. The Latin term "radius," which may also be used to describe a chariot wheel spoke, is where the word "radius" first appeared.

The length of tangents drawn from an external point is known to be constant. The circle's radius across the point of contact and any other point on the circle are perpendicular to the tangent. A quadrilateral has 360° of angles total. In light of this, 137° is the indicated angle measurement.

To learn more about radius from the given link

https://brainly.com/question/27696929

#SPJ4

Correct question is

Find the angle measure indicated. Assume that lines which appear to be tangent are tangent.

The polynomial â 2x2 + 700x represents the budget surplus of the town of Alphaville.


Betaville's surplus is represented by x2 - 100x + 80,000. If x represents the tax revenue in


thousands from both towns, enter the expression that represents the total surplus of both


towns together.


The expression that represents the total surplus of both towns together is ?

Answers

The total surplus of both towns together is represented by the polynomial [tex]3x^2 + 600x + 80,000.[/tex]

The expression that represents the total surplus of both towns together is (â 2x2 + 700x) + (x2 - 100x + 80,000).?

To find the total surplus of both towns together, we need to add the budget surplus of Alphaville and Betaville.

The budget surplus of Alphaville is represented by the polynomial [tex]2x^2 + 700x.[/tex]

The budget surplus of Betaville is represented by the polynomial x^2 - 100x + 80,000.

Therefore, the expression that represents the total surplus of both towns together is:

[tex](2x^2 + 700x) + (x^2 - 100x + 80,000)[/tex]

Simplifying this expression, we get:

[tex]3x^2 + 600x + 80,000[/tex]

Learn mora about polynomial,

brainly.com/question/20121808

#SPJ11

54 371 -5 -4 -3 -2 -1 0 -1 -2 -3 -4 -5 1 2 3 4 5 What is the equation of the blue line? X​

Answers

We can see here that the equation of the blue line is: y = -x -1.

What is equation?

Finding the value(s) of the variable(s) that make the equation true is the aim of an equation. These numbers are referred to be the equation's roots or solutions.

Finding an equation's answers may require algebraic manipulation, substitution, factoring, or other techniques, depending on how difficult the problem is.

The two points can be seen as thus:

(-1, 0) (0, -1)

Slope = -1.

Suppose, y = -x + a       (1, 0)

0 = - (-1) + a

a = -1

Thus, the equation is y = -x -1.

The complete question is attached in given image below.

Learn more about equation on https://brainly.com/question/29174899

#SPJ1

Use the properties of logarithms to simplify as much as possible. 3) In(4x^5) – In (x^3)- In 4 4) The price of beef has inflated by 2%. If the price of beef inflates 2% compounded biannually, how lung will it take for the price of beef to triple?

Answers

3) The expression In(4x^5) - In(x^3) - In 4 can be simplified using the properties of logarithms. We know that ln(a) - ln(b) = ln(a/b) and ln(a^n) = n ln(a), so we can write:In(4x^5) - In(x^3) - In 4 = In[(4x^5)/(x^3)] - In 4= In(4x^2) - In 4= In(4x^2/4)= In(x^2)Thus, the simplified expression is In(x^2).4) To solve this problem, we need to use the formula for compound interest:A = P(1 + r/n)^(nt)where A is the final amount, P is the initial amount, r is the interest rate (as a decimal), n is the number of times interest is compounded per year, and t is the number of years.We want to find t when A = 3P and r = 0.02 (since the price of beef has inflated by 2%). We are told that interest is compounded biannually, so n = 2. Plugging in these values and solving for t, we get:3P = P(1 + 0.02/2)^(2t)3 = (1.01)^2tln(3) = ln(1.01^2t)ln(3) = 2t ln(1.01)t = ln(3) / (2 ln(1.01))Using a calculator, we find t ≈ 34.64 years. Therefore, it will take about 34.64 years for the price of beef to triple at a 2% biannual inflation rate.

For more similar questions on topic Logarithmic functions.

https://brainly.com/app/ask?q=Logarithmic+functions.

#SPJ11

It will take approximately 110 years for the price of beef to triple when inflating 2% compounded biannually.

3) To simplify the expression In(4x^5) - In(x^3) - In(4), we will use the properties of logarithms:

- In(a) - In(b) = In(a/b)
- In(a^b) = b * In(a)

So, we can rewrite the expression as:

In(4x^5 / (x^3 * 4))

Now, we can simplify the expression inside the natural logarithm:

(4x^5) / (4x^3) = x^(5-3) = x^2

Thus, the simplified expression is:

In(x^2)

4) To find how long it will take for the price of beef to triple when inflating 2% compounded biannually, we can use the compound interest formula:

A = P(1 + r/n)^(nt)

where A is the final amount, P is the initial amount, r is the annual interest rate, n is the number of times interest is compounded per year, and t is the time in years. In this case, we want the final amount to be triple the initial amount:

3P = P(1 + 0.02/2)^(2t)

To solve for t, we can divide both sides by P:

3 = (1 + 0.01)^(2t)

Now, take the natural logarithm of both sides and use the properties of logarithms:

ln(3) = ln((1 + 0.01)^(2t))
ln(3) = 2t * ln(1 + 0.01)

Finally, isolate t:

t = ln(3) / (2 * ln(1 + 0.01))

t ≈ 109.96

It will take approximately 110 years for the price of beef to triple when inflating 2% compounded biannually.

To learn more about compound interest, refer below:

https://brainly.com/question/14295570

#SPJ11

A satellite orbiting Earth travels 4.95×10^7 meters for each orbit. It takes 6,600 seconds to make one orbit. What is the speed of the satellite in meters per second? Give your answer in standard form.

Answers

The value of the speed of the satellite in meters per second is,

⇒ Speed = 7.5 x 10³ m /sec

We have to given that;

A satellite orbiting Earth travels 4.95×10⁷ meters for each orbit.

And, It takes 6,600 seconds to make one orbit.

Hence, the speed of the satellite in meters per second is,

⇒ Speed = Distance / Time

⇒ Speed = 4.95×10⁷/ 6,600

⇒ Speed = 0.00075 x 10⁷

⇒ Speed = 7.5 x 10⁻⁴ x 10⁷

⇒ Speed = 7.5 x 10³ m /sec

Thus, The value of the speed of the satellite in meters per second is,

⇒ Speed = 7.5 x 10³ m /sec

Learn more about the scientific notation visit:

brainly.com/question/1767229

#SPJ1

If the area around the cylinder is 64 cm² and the area of the top is 16 cm², what is the surface area of the cylinder?
help me please lol

Answers

The surface area of the cylinder is 96cm²

Calculating the surface area of the cylinder?

From the question, we have the following parameters that can be used in our computation:

The area around the cylinder is 64 cm² The area of the top is 16 cm²

Using the above as a guide, we have the following:

Surface area of the cylinder = The area around the cylinder + 2 * The area of the top

Substitute the known values in the above equation, so, we have the following representation

Surface area of the cylinder = 64 + 2 * 16

Evaluate

Surface area of the cylinder = 96

Hence, the surface area of the cylinder is 96cm²

Read mroe about surface area at

https://brainly.com/question/26403859

#SPJ1

Which is a reasonable estimate for the difference 5 1/2- 3 5/9? Circle
the letter of the correct answer
A between 1/2 and 1
B between 1 and 1 1/2
C between 1 1/2 and 2
D between 2 and 2 1/2
Elise chose D as the correct answer. How did she get that answer?

Answers

Step-by-step explanation:

To estimate the difference between 5 1/2 and 3 5/9, we can first round the fractions to the nearest whole number or simpler fractions. In this case, we can round 1/2 to 1/2 and 5/9 to 1/2 as well. Now, we have:

5 1/2 − 3 1/2

Subtracting the whole numbers, we get:

5−3=2

Subtracting the fractions, we get:

1/2 − 1/2 = 0

So, the estimated difference is 2, which falls between 2 and 2 1/2. Therefore, Elise chose option D as the correct answer.

Find the area of the shaded region. Round your answer to the nearest hundredth.

Answers

Answer:

The radius of the circle is 5/√2 = (5√2)/2 inches.

Area of circle = π((5√2)/2)^2

= 25π/2 square inches

Area of triangle = (1/2)(5√2)((5√2)/2)

= 25/2 square inches

Area of shaded region

= (25/2)(π - 1) = 26.77 square inches

The dean of students at a large college is interested in learning about their opinions regarding the percentage of


first-year students who should be given parking privileges in the main lot. He sends out an email survey to all


students about this issue. A large number of first-year students reply but very few sophomores, juniors, and seniors


reply. Based on the responses he receives, he constructs a 90% confidence interval for the true proportion of


students who believe first-year students should be given parking privileges in the main lot to be (0. 71, 0. 79). Which


of the following may have an impact on the confidence interval, but is not accounted for by the margin of error?


O response bias


O nonresponse bias


O sampling variation


O undercoverage bias


Mark this and retum


Save and Exit


Next


Submit

Answers

b. Nonresponse bias creates an impact on the confidence interval, but is not accounted for by the margin of error.

Given that, the dean of students at a large college is interested in learning about the opinions of students regarding the percentage of first-year students who should be given parking privileges in the main lot. He sends out an email survey to all students about this issue, but receives very few responses from sophomores, juniors, and seniors. Based on the responses he receives, he constructs a 90% confidence interval for the true proportion of students who believe first-year students should be given parking privileges in the main lot to be (0.71, 0.79).

Response bias refers to a systematic pattern of incorrect responses in a survey, which can be caused by factors such as question wording, social desirability bias, or interviewer bias.

Nonresponse bias, on the other hand, occurs when individuals who do not respond to a survey are systematically different from those who do respond, leading to a biased estimate of the population parameter.

Sampling variation refers to the fact that different samples from the same population can yield different estimates of the population parameter due to random variation.

Under coverage bias occurs when some members of the population are systematically excluded from the sample, leading to a biased estimate of the population parameter.

In this scenario, the fact that very few sophomores, juniors, and seniors responded to the survey could potentially introduce nonresponse bias, since those who did respond may not be representative of the entire population of students.

However, the confidence interval itself does not account for nonresponse bias or any other sources of bias. Instead, it reflects the range of values that is likely to contain the true proportion of students who believe first-year students should be given parking privileges in the main lot, based on the data that was collected.

To know more about Confidence Interval here

https://brainly.com/question/29680703

#SPJ4

12. A normal distribution has a mean of 34 and a standard deviation of 7. Find the range of
values that represent the middle 95% of the data.

F. 27 G. 20 X 48
H. 13 J. 6

Answers

The range of values that represent the middle 95% of the data is from 20.18 to 47.82 or (20.18, 47.82).

What is Hypothesis test?

A measurable speculation test is a strategy for factual deduction used to conclude whether the information within reach adequately support a specific speculation. We can make probabilistic statements about the parameters of the population thanks to hypothesis testing.

According to question:

The middle 95% of a normal distribution is located within 1.96 standard deviations from the mean in both directions.

Therefore, the lower limit is:

34 - 1.96(7) = 20.18

And the upper limit is:

34 + 1.96(7) = 47.82

So the range of values that represent the middle 95% of the data is from 20.18 to 47.82 or (20.18, 47.82).

To know more about Mean visit:

brainly.com/question/30094057

#SPJ1

Find the area k of the trianglea = 3, c = 2, b = 135 degrees

Answers

The area k of the triangle a = 3, c = 2, b = 135 degrees is approximately 1.06125 square units.

The area k of the triangle, we can use the formula:
k = (1/2) * b * c * sin(A)
where A is the angle opposite side a.
Find A, we can use the fact that the angles in a triangle add up to 180 degrees:
A + B + C = 180
Substituting in the given values, we get:
A + 135 + 180 = 360
A = 45 degrees
Now we can plug in all the values into the area formula:
k = (1/2) * 2 * 3 * sin(45)
k = 1.5 * 0.707
k = 1.06125
Therefore, the area k of the triangle is approximately 1.06125 square units.

Read more about triangle.

https://brainly.com/question/22964077

#SPJ11

In triangle abc, point d is on side ac, ab=bd=dc=12 inches, and measurement of angle bdc= 2 times the measurement of angle abd. find ac

Answers

The length of AC in triangle ABC is 24 inches.

In triangle ABC, let point D be on side AC such that AB = BD = DC = 12 inches. We are given that the measure of angle BDC is twice the measure of angle ABD.

To find the length of AC, we can use the Law of Cosines. This law relates the lengths of the sides of a triangle to the cosine of one of its angles. In this case, we can apply the Law of Cosines to triangle ABD to find the length of AD:

AD^2 = AB^2 + BD^2 - 2 * AB * BD * cos(ABD)

Since AB = BD = 12 inches, we have:

AD^2 = 12^2 + 12^2 - 2 * 12 * 12 * cos(ABD)

AD^2 = 288 - 288 * cos(ABD)

Now, let's consider triangle BDC. We are given that the measure of angle BDC is twice the measure of angle ABD. Let's denote the measure of angle ABD as x. Therefore, the measure of angle BDC is 2x.

Since the sum of angles in a triangle is 180 degrees, we can write:

x + 2x + angle BCD = 180

3x + angle BCD = 180

angle BCD = 180 - 3x

Now, let's apply the Law of Cosines to triangle BDC to find the length of BC:

BC^2 = BD^2 + CD^2 - 2 * BD * CD * cos(BDC)

BC^2 = 12^2 + 12^2 - 2 * 12 * 12 * cos(2x)

BC^2 = 288 - 288 * cos(2x)

Since AD = DC, we have AD = 12 inches. Now we can write the equation for the total length AC:

AC = AD + DC

AC = 12 + 12

AC = 24 inches

Therefore, the length of AC in triangle ABC is 24 inches.

Learn more about triangle,

https://brainly.com/question/28470545

#SPJ11

1. A tree was cut down. What 3D shape does it closely resemble?


A. Prism


B. Pyramid


C. Cylinder


D. Cone


2. A ball with radius 5. 5 cm fits tightly inside a cube. Find the volume of the


unoccupied space inside the cube. Round to the nearest cm.


A. 531


B. 166


C. 697


D. 634

Answers

The volume of the unoccupied space inside the cube is the volume of the cube minus the volume of the ball, which is approximately 166.

1. D. Cone. When a tree is cut down, its trunk typically has a roughly cylindrical shape with a tapered end, which closely resembles a cone.

2. B. 166. The diameter of the ball is 11 cm, which is also the length of the diagonal of the cube. Let's call the side length of the cube "s". Then, we can use the Pythagorean theorem to find s:

s^2 + s^2 + s^2 = 11^2

3s^2 = 121

s^2 = 121/3

The volume of the cube is s^3, which is approximately 166. The volume of the ball is (4/3)πr^3, where r is the radius of the ball. Since the ball fits tightly inside the cube, its diameter is equal to the side length of the cube, which is s√3. Thus, r = (s√3)/2 - 5.5.

The volume of the unoccupied space inside the cube is the volume of the cube minus the volume of the ball, which is approximately 166.

To know more about  volume of the cube refer here:

https://brainly.com/question/23526372

#SPJ11

Find the inverse of y=(2/3)x^5-10

Answers

The inverse of the function y= (2/3)x^5-10 is y = [3/2(x + 10)]^1/5

Finding the inverse of the function

From the question, we have the following parameters that can be used in our computation:

y= (2/3)x^5-10

Swap the ocurrence of x and y

so, we have the following representation

x = (2/3)y^5-10

Next, we have

(2/3)y^5 = x + 10

This gives

y^5 = 3/2(x + 10)

Take the fifth root of both sides

y = [3/2(x + 10)]^1/5

Hence, the inverse function is y = [3/2(x + 10)]^1/5

Read more about inverse function at

https://brainly.com/question/3831584

#SPJ1

Let f(x) = x² – 6x. Round all answers to 2 decimal places. = a. Find the slope of the secant line joining (2, f(2) and (7, f(7)). Slope of secant line = b. Find the slope of the secant line joining (6, f(6)) and (6 + h, f(6 + h)). Slope of secant line = c. Find the slope of the tangent line at (6, f(6)). Slope of the tangent line d. Find the equation of the tangent line at (6, f(6)). y =

Answers

The equation of the tangent line at (6, f(6)) is y = 6x - 48.

a. The slope of the secant line joining (2, f(2)) and (7, f(7)) is:

slope = (f(7) - f(2)) / (7 - 2)

We can find f(7) and f(2) by plugging in x = 7 and x = 2 into the expression for f(x):

f(7) = 7² - 6(7) = 7

f(2) = 2² - 6(2) = -8

Substituting these values into the slope formula, we get:

slope = (7 - (-8)) / (7 - 2) = 3

Therefore, the slope of the secant line joining (2, f(2)) and (7, f(7)) is 3.

b. The slope of the secant line joining (6, f(6)) and (6 + h, f(6 + h)) is:

slope = (f(6 + h) - f(6)) / ((6 + h) - 6) = (f(6 + h) - f(6)) / h

We can find f(6) and f(6 + h) by plugging in x = 6 and x = 6 + h into the expression for f(x):

f(6) = 6² - 6(6) = -12

f(6 + h) = (6 + h)² - 6(6 + h) = h² - 6h + 36 - 36 - 6h = h² - 12h

Substituting these values into the slope formula, we get:

slope = (h² - 12h - (-12)) / h = h - 12

Therefore, the slope of the secant line joining (6, f(6)) and (6 + h, f(6 + h)) is h - 12.

c. The slope of the tangent line at (6, f(6)) is the derivative of f(x) at x = 6:

f'(x) = 2x - 6

f'(6) = 2(6) - 6 = 6

Therefore, the slope of the tangent line at (6, f(6)) is 6.

d. To find the equation of the tangent line at (6, f(6)), we use the point-slope form of a line:

y - f(6) = f'(6)(x - 6)

Substituting f(6) and f'(6) into this equation, we get:

y - (-12) = 6(x - 6)

Simplifying, we get:

y = 6x - 48

Therefore, the equation of the tangent line at (6, f(6)) is y = 6x - 48.

To learn more about tangent line visit:https://brainly.com/question/31326507

#SPJ11

On the Employee Sales Summary sheet, the function used to add together the last employee's sales for the three months is ___________________.
Group of answer choices
=SUM(E16)
=E16+E16+E16
=SUM('Employee Sales October:Employee Sales December'!E16)
=SUM('Employee Sales January:Employee Sales March'!E5)

Answers

The function used to add together the last employee's sales for the three months on the Employee Sales Summary sheet is: =SUM('Employee Sales January:Employee Sales March'!E5)

SUM(E16): This function adds up the values in cells E16 from the current sheet. If the last employee's sales for the three months are stored in cells E16, E17, and E18, then this function would correctly calculate the total.

E16+E16+E16: This expression adds up the value in cell E16 three times. If the last employee's sales for the three months are stored in cells E16, E17, and E18, then this expression would not calculate the total correctly.

SUM('Employee Sales October:Employee Sales December'!E16): This function adds up the values in cell E16 from all sheets between Employee Sales October and Employee Sales December (inclusive). If the last employee's sales for the three months are stored in cells E16, E17, and E18 on different sheets, then this function could be used to calculate the total.

SUM('Employee Sales January:Employee Sales March'!E5): This function adds up the values in cell E5 from all sheets between Employee Sales January and Employee Sales March (inclusive).

Learn more about employee's sales

https://brainly.com/question/11343692

#SPJ4

Full Question: On the Employee Sales Summary sheet, the function used to add together the last employee's sales for the three months is ___________________.

Group of answer choices

=SUM(E16)=E16+E16+E16=SUM('Employee Sales October:Employee Sales December'!E16)=SUM('Employee Sales January:Employee Sales March'!E5)

Use the rules to find derivatives of the following functions at the specified values
h(x) = 8x at x = 4
h'(4) = _____

Answers

To find the derivative of h(x) = 8x, we use the power rule, which states that the derivative of x^n is nx^(n-1). Applying this rule to h(x), we get h'(x) = 8.

To find the value of h'(4), we simply plug in x = 4 into our derivative expression: h'(4) = 8.

Therefore, the derivative of h(x) = 8x at x = 4 is h'(4) = 8.
To find the derivative of the function h(x) = 8x at x = 4, you can use the power rule for differentiation. The power rule states that if h(x) = x^n, then h'(x) = n * x^(n-1).

For h(x) = 8x, n = 1, so:

h'(x) = 1 * 8x^(1-1) = 8

Now, to find h'(4), just plug in x = 4:

h'(4) = 8

So, h'(4) = 8.

Visit here to learn more about derivative brainly.com/question/30365299

#SPJ11

10% of people are left handed. If 800 people are randomly selected, find the likelihood that at least 12% of the sample is left handed

Answers

The likelihood of at least 12% of the sample being left-handed is approximately 0.007 or 0.7%.

Let X be the number of left-handed people in a sample of 800 individuals. Since the probability of a person being left-handed is 0.1, the probability of a person being right-handed is 0.9. Then, X follows a binomial distribution with n = 800 and p = 0.1.

P(X ≥ 0.12*800) = P(X ≥ 96)

where 96 is the smallest integer greater than or equal to 0.12*800.

[tex]P(X > =96)-P(X < 96)=1-[K=0 to 95](800 CHOOSE )(0.1^{k} (0.9)^{2} (800-k)[/tex]

This is the complement of the probability of getting less than 96 left-handed people in the sample. Using a calculator or statistical software, we can find that:

P(X ≥ 96) ≈ 0.007

To know more about binomial distribution:

https://brainly.com/question/31197941

#SPJ4

Cost, revenue, and profit are in dollars and x is the number of units. If the marginal cost for a product is MC = 8x + 70 and the total cost of producing 30 units is $6000, find the cost of producing 40 units. $ Need Help? Watch Talk to a Tutor Read it MY NOTE Cost, revenue, and profit are in dollars and x is the number of units. A firm knows that its marginal cost for a product is MC - 4x + 25, that its marginal revenue is MR - 55 - 6x, and that the cost of production of 80 units is $14,920. (a) Find the optimal level of production. units (b) Find the profit function. P(x) = (c) Find the profit or loss at the optimal level. There is a of $ -Select-

Answers

The profit is positive, the firm makes a profit of $21,243 at the optimal level of production.

Cost of producing 40 units

We know that the total cost of producing 30 units is $6000. Let's denote the total cost function by C(x), where x is the number of units produced. Then, we have:

C(30) = $6000

The marginal cost function is given as MC = 8x + 70. Integrating this function, we get the total cost function as:

C(x) = [tex]4x^2[/tex] + 70x + C

To find the value of the constant C, we use the fact that C(30) = $6000:

4[tex](30)^2[/tex] + 70(30) + C = $6000

Solving for C, we get:

C = $300

Therefore, the total cost function is:

C(x) = [tex]4x^2[/tex] + 70x + $300

To find the cost of producing 40 units, we evaluate C(40):

C(40) = [tex]4(40)^2[/tex] + 70(40) + $300

C(40) = $7000

Therefore, the cost of producing 40 units is $7000.

Optimal level of production:

The optimal level of production is the value of x that maximizes the profit function. To find this value, we need to set the marginal cost equal to the marginal revenue:

MC = MR

8x + 70 = -6x + 55

Solving for x, we get:

x = 5/7

Since the optimal level of production should be a whole number, we round x up to 1 unit.

Therefore, the optimal level of production is 1 unit.

Profit function:

The profit function is given as:

P(x) = R(x) - C(x)

where R(x) is the revenue function and C(x) is the cost function.

The marginal revenue function is given as MR = -6x + 55. Integrating this function, we get the revenue function as:

R(x) = -[tex]3x^2[/tex] + 55x + D

To find the value of the constant D, we use the fact that the revenue at x = 80 is $14,920:

[tex]-3(80)^2[/tex] + 55(80) + D = $14,920

Solving for D, we get:

D = $21,520

Therefore, the revenue function is:

R(x) = -[tex]3x^2[/tex] + 55x + $21,520

Substituting the cost function and revenue function in the profit function, we get:

P(x) = ([tex]-3x^2[/tex] + 55x + $21,520) - (4x^2 + 25x + $300)

Simplifying, we get:

P(x) = -[tex]7x^2[/tex] + 30x + $21,220

Therefore, the profit function is P(x) = [tex]-7x^2[/tex] + 30x + $21,220.

Profit or loss at the optimal level:

To find the profit or loss at the optimal level, we evaluate the profit function at x = 1:

P(1) = [tex]-7(1)^2[/tex] + 30(1) + $21,220

P(1) = $21,243

Since the profit is positive, the firm makes a profit of $21,243 at the optimal level of production.

Learn more about marginal cost

https://brainly.com/question/7781429

#SPJ4

Gross Monthly Income: Jackson works for a pipe line company and is paid $18. 50 per hour. Although he will have overtime, it is not guaranteed when or where, so Jackson will only build a budget on 40 hours per week. What is Jackson’s gross monthly income for 40 hours per week? Type in the correct dollar amount to the nearest cent. Do not include the dollar sign or letters.


A. Gross Annual Income: $


B. Gross Monthly Income: $

Answers

Jackson's gross monthly income for 40 hours per week is approximately $3,201.70 and gross annual income s $38,480.

To find Jackson's gross monthly income, we first need to find his gross weekly income.

Jackson's hourly wage is $18.50, so his weekly gross income for 40 hours of work is:

40 hours/week x $18.50/hour = $740/week

Calculate annual income:

To determine the gross annual income, we need to consider how many weeks there are in a year. Assuming 52 weeks in a year:

Annual income = Weekly income * Number of weeks in a year

Annual income = $740 * 52 = $38,480

To find Jackson's gross monthly income, we can multiply his weekly gross income by the number of weeks in a month (approximately 4.33):

$740/week x 4.33 weeks/month ≈ $3,201.70/month

Therefore, Jackson's gross monthly income for 40 hours per week is approximately $3,201.70.

To know more about gross monthly income, visit:

https://brainly.com/question/30617016#

#SPJ11

Liquid a has a density of 1. 2 g/cm'
150 cm of liquid a is mixed with some of liquid b to make liquid c.
liquid c has a mass of 220 g and a density of 1. 1 g/cm
find the density of liquid b.

Answers

If Liquid a has a density of 1. 2 g/cm³, 150 cm of Liquid a is mixed with some of Liquid b to make Liquid c whose mass is 220 g and has a density of  1.1 g/cm³, then the density of liquid B is 0.8 g/cm³.

To find the density of liquid B, you can follow these steps:
1. Calculate the mass of liquid A using its density and volume:
Liquid A has a density of 1.2 g/cm³ and a volume of 150 cm³.

Mass of A = Density of A × Volume of A = 1.2 g/cm³ × 150 cm³ = 180 g

2. Calculate the mass of liquid B using the mass of liquid C and mass of liquid A:
Liquid C has a mass of 220 g.
Mass of B = Mass of C - Mass of A = 220 g - 180 g = 40 g

3. Calculate the volume of liquid C using its mass and density:
Liquid C has a density of 1.1 g/cm³.
Volume of C = Mass of C ÷ Density of C = 220 g ÷ 1.1 g/cm³ = 200 cm³

4. Calculate the volume of liquid B using the volume of liquid C and the volume of liquid A:
Volume of B = Volume of C - Volume of A = 200 cm³ - 150 cm³ = 50 cm³

5. Calculate the density of liquid B using it's mass and volume:
Density of B = Mass of B ÷ Volume of B = 40 g ÷ 50 cm³ = 0.8 g/cm³

So, the density of liquid B is 0.8 g/cm³.

Learn more about Density:

https://brainly.com/question/1354972

#SPJ11

Which graph represents the function f {x} = -log (x-1) + 1?

Graph A
Graph B
Graph C
Graph D

Answers

The correct answer is graph D.

Sam makes mini pancakes for breakfast. Each pancake is a circle with a diameter of 6 centimeters.

a) Calculate the circumference of each pancake.

b) Calculate the area of each pancake.

Answers

Answer:

a) 18.84 cm

b) 28.26 cm²

Step-by-step explanation:

We Know

Each pancake is a circle with a diameter of 6 centimeters.

a)  Calculate the circumference of each pancake.

Circumference of circle = d · π

d = 6 cm

We Take

6 · 3.14 = 18.84 cm

So, the circumference of each pancake is 18.84 cm.

b) Calculate the area of each pancake.

Area of circle = r² · π

r = 1/2 · d

r = 1/2 · 6 = 3 cm

We Take

3² · 3.14 = 28.26 cm²

So, the area of each pancake is 28.26 cm².

If you charge more than your limit in your credit card

Answers



If you charge more than your limit on your credit card, you will likely face several consequences, including:

1. Over-limit fees: Many credit card companies charge a fee if you exceed your credit limit. This fee can vary depending on the issuer, but it's typically around $25 to $35.

2. Increased interest rates: If you go over your credit limit, your credit card company may increase your interest rate, making it more expensive to carry a balance on your card.

3. Lower credit score: Exceeding your credit limit can negatively impact your credit score, as it's an indication of risky financial behavior.

4. Reduced credit availability: Your credit card company may reduce your credit limit if you consistently go over the limit, making it harder for you to access credit in the future.

5. Declined transactions: If you're significantly over your limit, your credit card company may start declining transactions to prevent further over-limit spending.

To avoid these consequences, it's important to monitor your spending and ensure you stay within your credit limit. If you find yourself consistently approaching your limit, consider requesting a credit limit increase or using other methods to manage your spending.

To know more about "Credit cards" refer here:

https://brainly.com/question/31598744#

#SPJ11

Ariana has 144 peaches. She has to pack 9 boxes with an equal number of peaches. How many peaches should she pack in each box.

Answers

Answer:

16 peaches

Step-by-step explanation:

Let's break this down:

Total: 144 peaches

Number of boxes she has to fill evenly: 9

Question: How many peaches are able to fit into each box evenly?

144 peaches/9 boxes = 16 peaches

So, Ariana should pack 16 peaches into each box

Hope this helps :)

1 ml =

a

litres


ii)

b

ml = 1 litre


iii) 1 cl =

c

litres


iv)

d

cl = 1 litre


v) 1 cl =

e

ml


vi)

f

cl = 1 ml

Answers

The corresponding measure of the parameters are;

i. 1ml = 0. 001 liter a.

ii. 1000ml = 1 liter b.

iii. 1 cl = 0. 01 liter c.

iv. 10dcl = 1 liter d.

v. 1cl = 100ml e.

v. 0. 01 cl = 1ml f.

How to determine the values

To convert the factors, we need to know the following conversion rates.

We have;

1 milliliter = 0. 001 liter

1 centiliter = 0. 01 liter

1 deciliter = 0. 1 liter

1 cubic centimeter = 1 millimeter

Hence, the sizes are determined by the corresponding factor.

Learn more about conversion factors at: https://brainly.com/question/97386

#SPJ4

Complete question:

Convert the following to their equivalent measurement for each letter

i. 1 ml = a liters

ii) b ml = 1 liters

iii) 1 cl = c liters

iv)d cl = 1 liters

v) 1 cl = e ml

vi) f cl = 1 ml

+
ent will
A circle with center (7,3) and radius of 5 is graphed below with a square inscribed in
the circle.
+
Dillon says to write the equation of the tangent line you need the opposite-reciprocal
slope of the slope of the radius and Chelsey says you need to use the same slope as
the radius. Who is correct and why? Write the equation of the tangent line.
Part B: Find the perimeter of BCDE.
Part C: Find the area of BCDE.
Part D: Prove BCDE is a square.

Answers

Chelsey is correct that we use the same slope as the radius to write the equation of the tangent line, even though the slope of the radius is undefined at the points of tangency.

The perimeter will be 20 ✓2 units.

The area will be 50 units²

How to explain the information

Chelsey is correct, and the reason is that a tangent line to a circle at a given point is always perpendicular to the radius of the circle at that point. This means that the slope of the tangent line and the slope of the radius at the point of tangency are negative reciprocals of each other.

Chelsey is correct that we use the same slope as the radius to write the equation of the tangent line.

The perimeter will be:

= 4 × BC

= 20 ✓2

The area will be:

= BC²

= (5✓2)²

= 50

It should be noted that BCDE is a square as EBC is 90°.

Learn more about slope on

https://brainly.com/question/3493733

#SPJ1

Other Questions
GEOMETRY URGENT HELP: SINE, COSINE TANGENTplease help me i have no idea what i am doing What is the volume occupied by 3.67 moles of h2 gas at stp? (hint: you do not need the molar mass to do this conversion because it is a gas) * A rock is placed on a scale and gives a reading of 76.89 grams. The rock is then placed in a graduated cylinder with 63.12 mL of water, the water rises to a volume of 73.54mL What is the density of the rock? (you answer must have a total of 2 decimals) compute (7 4/9 -8)*3.6-1.6*(1/3-3/4)+ 1 2/5 (0.35) Please help !!!!! !!!!! Once all of his loans are paid off, what will hals total lifetime cost be? round all dollar values to the nearest cent. a. $9,023.28 b. $8,467.20 c. $11,498.40 d. $13,615.20 Complete the code for the provided Sequence class. Your code should provide the expected results when run through the provided SequenceTester class. Add code for these two methods in the Sequence class: public Sequence append(Sequence other) append creates a new sequence, appending this and the other sequence, without modifying either sequence. For example, if a is 1 4 9 16 and b is the sequence 9 7 4 9 11 then the call a. Append(b) returns the sequence 1 4 9 16 9 7 4 9 11 without modifying a or b. Public Sequence merge(Sequence other) merges two sequences, alternating elements from both sequences. If one sequence is shorter than the other, then alternate as long as you can and then append the remaining elements from the longer sequence. For example, if a is 1 4 9 16 and b is 9 7 4 9 11 then a. Merge(b) returns the sequence without modifying a or b. 3. 1 9 4 7 9 4 16 9 11 how Perseus feels when he first gazes on the sleeping Medusa, as well as how and why these feelings change. 2. Net present value (NPV)The capital budgeting process is comprehensive and is based on certain assumptions, models, and benchmarks. This process often begins with a project analysis. Generally, the first step in a capital budgeting project analysiswhich occurs before any evaluation method is appliedinvolves estimating the _____________. Evaluating cash flows with the NPV methodThe net present value (NPV) rule is considered one of the most common and preferred criteria that generally lead to good investment decisions. Consider this case:Suppose Blue Hamster Manufacturing Inc. Is evaluating a proposed capital budgeting project (project Alpha) that will require an initial investment of $600,000. The project is expected to generate the following net cash flows:YearCash FlowYear 1 $350,000Year 2 425,000Year 3 450,000Year 4 475,000Blue Hamster Manufacturing Inc. s weighted average cost of capital is 7%, and project Alpha has the same risk as the firms average project. Based on the cash flows, what is project Alphas net present value (NPV)? (Note: Do not round your intermediate calculations. )$828,024$228,024$993,629$1,278,024Making the accept or reject decisionBlue Hamster Manufacturing Inc. s decision to accept or reject project Alpha is independent of its decisions on other projects. If the firm follows the NPV method, it should____ project Alpha. Suppose your boss has asked you to analyze two mutually exclusive projectsproject A and project B. Both projects require the same investment amount, and the sum of cash inflows of Project A is larger than the sum of cash inflows of project B. A coworker told you that you dont need to do an NPV analysis of the projects because you already know that project A will have a larger NPV than project B. Do you agree with your coworkers statement?No, the NPV calculation is based on percentage returns, so the size of a projects cash flows does not affect a projects NPV. Yes, project A will always have the largest NPV, because its cash inflows are greater than project Bs cash inflows. No, the NPV calculation will take into account not only the projects cash inflows but also the timing of cash inflows and outflows. Consequently, project B could have a larger NPV than project A, even though project A has larger cash inflows The _____ style of communication is characterized by language that requires the listener to carefully note what is being said and to observe how the sender is presenting the message what is the term that is often used for the acquisition of services? what is the term that is often used for the acquisition of services? contracting requisitioning ordering purchasing IELTS Two paragraphs First paragraph specific details about menSecond paragraph specific details about women Typical household bleach has a ph of 13. what is the h3o concentration in household bleach? 1.text, graphics,and animators are blank elements.2.Lines,symbols, color,gaze, texture,and framing are blank.3.video is a technology of capturing blank, transmitting,and reconstructing moving picture.4.multimedia is a combination of multiple forms of media useful to blank.5.color has the blank effect on our emotions.6.animation is a collection of graphics applied with blank.7.visual elements are any characteristics that we can blank.8.black represents evil, mysterious,power, and blank.9.symbol are blank of concept or ideas.10.blank conveys information to the audience,tell stories and describe thingplease answer it (g) two masses mand m2(m > m2) slide down a rough inclined surface of the samelength and inclination. which of the masses would be the first to get to the bottom? givereasons for your answer. According to the AAA Foundation for Traffic Safety's American Driving Survey, U. S. Drivers spend,on average, 51 minutes behind the wheel each day. A researcher believes this is an overstatement. To investigate, a random sample of 75 drivers were selected. The study revealed that the meantime behind the wheel for the sample of 75 drivers was 46. 4 minutes with a standard deviation of18. 8 minutes. Is there convincing evidence that the mean time behind the wheel for all U. S. Driversis less than 51 minutes? Use a = 0. 1. Yosef is so intrigued by what he sees in his microscope that he walks to a nearby pond and collects a sample of pond water to learn more about paramecium. he uses a single drop of the pond water to create a wet mount slide, which allows him to see living paramecia moving around in the water, unlike the processed and dyed paramecia in the prepared slide (part a). how do the structures observed under magnification help the organism carry out basic functions of life? write your findings in the space provided. If sodium increases in the ecf, water will move from:. What is CPR? How is it used?What is First Aid?What is an AED? How is it used? Question 10(Multiple Choice Worth 2 points)(Comparing Data MC)The box plots display measures from data collected when 20 people were asked about their wait time at a drive-thru restaurant window.A horizontal line starting at 0, with tick marks every one-half unit up to 32. The line is labeled Wait Time In Minutes. The box extends from 8.5 to 15.5 on the number line. A line in the box is at 12. The lines outside the box end at 3 and 27. The graph is titled Super Fast Food.A horizontal line starting at 0, with tick marks every one-half unit up to 32. The line is labeled Wait Time In Minutes. The box extends from 9.5 to 24 on the number line. A line in the box is at 15.5. The lines outside the box end at 2 and 30. The graph is titled Burger Quick.Which drive-thru typically has more wait time, and why? Burger Quick, because it has a larger median Burger Quick, because it has a larger mean Super Fast Food, because it has a larger median Super Fast Food, because it has a larger meanQuestion 11(Multiple Choice Worth 2 points)(Creating Graphical Representations MC)The number of carbohydrates from 10 different tortilla sandwich wraps sold in a grocery store was collected.Which graphical representation would be most appropriate for the data, and why? Circle chart, because the data is categorical Line plot, because there is a large set of data Histogram, because you can see each individual data point Stem-and-leaf plot, because you can see each individual data pointQuestion 12(Multiple Choice Worth 2 points)(Comparing Data MC)The line plots represent data collected on the travel times to school from two groups of 15 students.A horizontal line starting at 0, with tick marks every two units up to 28. The line is labeled Minutes Traveled. There is one dot above 10, 16, 20, and 28. There are two dots above 8 and 14. There are three dots above 18. There are four dots above 12. The graph is titled Bus 14 Travel Times.A horizontal line starting at 0, with tick marks every two units up to 28. The line is labeled Minutes Traveled. There is one dot above 8, 9, 18, 20, and 22. There are two dots above 6, 10, 12, 14, and 16. The graph is titled Bus 18 Travel Times.Compare the data and use the correct measure of center to determine which bus typically has the faster travel time. Round your answer to the nearest whole number, if necessary, and explain your answer. Bus 14, with a median of 14 Bus 18, with a mean of 12 Bus 14, with a mean of 14 Bus 18, with a median of 12Question 13(Multiple Choice Worth 2 points)(Appropriate Measures MC)The line plot displays the number of roses purchased per day at a grocery store.A horizontal line starting at 1 with tick marks every one unit up to 10. The line is labeled Number of Rose Bouquets, and the graph is titled Roses Purchased Per Day. There is one dot above 1 and 2. There are two dots above 8. There are three dots above 6, 7, and 9.Which of the following is the best measure of variability for the data, and what is its value? The range is the best measure of variability, and it equals 8. The range is the best measure of variability, and it equals 2.5. The IQR is the best measure of variability, and it equals 8. The IQR is the best measure of variability, and it equals 2.5